I actually have to prove the following :
$\mathbf{NL} \subseteq \mathbf{NC_2}$
I have the following approach :
I will prove that $\mathbf{PATH} = \{〈D, s, t〉 | \text{D is a directed graph with a path from vertex s to t}\} \in \mathbf{NC_2}$.
I will show that $\mathbf{NC_2}$ is closed under log-space reductions i.e:
$$(1): B \in \mathbf{NC_2} \hbox{ and } A \le_l B \Longrightarrow A \in \mathbf{NC_2}$$
where $\le_l$ is the logspace reduction defined as
$$A \le_l B :\Longleftrightarrow (\exists M \hbox{ TM}, \forall x)[x \in A \Longleftrightarrow M(x) \in B]$$
($M$ is a TM which runs in logarithmic space).
- Since $\mathbf{PATH}$ is an $\mathbf{NL}$-complete problem the proof will be done.
Proving The 1st part was easy, i am stuck at the second part and have no idea how to proceed.
Any help?
-
$\begingroup$ What uniformity condition do you impose? $\endgroup$user12859– user128592015年11月05日 15:04:58 +00:00Commented Nov 5, 2015 at 15:04
-
$\begingroup$ Log-space uniformity! $\endgroup$Vikash Balasubramanian– Vikash Balasubramanian2015年11月05日 15:13:38 +00:00Commented Nov 5, 2015 at 15:13
-
$\begingroup$ Do you have to use the approach described in your question? $\endgroup$user12859– user128592015年11月05日 15:22:26 +00:00Commented Nov 5, 2015 at 15:22
-
$\begingroup$ Yes, i have seen another approach and i want to know if i can do it using mine. $\endgroup$Vikash Balasubramanian– Vikash Balasubramanian2015年11月05日 15:23:47 +00:00Commented Nov 5, 2015 at 15:23
-
$\begingroup$ The circuit should presumably solve a polynomial number of logspace problems to find M(x). $\:$ (Also, if you've gotten to the point of asking about a different proof strategy, then you might also want to strengthen it to show that NL is a subset of Dlogtime-uniform AC$_1$.) $\;\;\;\;$ $\endgroup$user12859– user128592015年11月05日 15:30:41 +00:00Commented Nov 5, 2015 at 15:30
1 Answer 1
We want to show that if $A\le_lB$ and $B\in NC_2$ then $A\in NC_2$.
Since $A\le_l B$ we have a Turing machine $M_{f,i}(x)$ calculating the i'th bit of the reduction output $f(x)_i,ドル using logarithmic space.
$M_{f,i}(x)=1 \iff \left( G_{M_{f,i}(x)},c_{start},c_{acc} \right)\in PATH$
Where $G_{M_{f,i}(x)}$ is the configuration graph of the computation of the logspace machine $M_{f,i}$ on $x,ドルand $c_{start},c_{acc}$ are the initial and accepting configurations, correspondingly.
Now your $NC_2$ circuit for $A$ will, "in parallel", compute all the bits of the reduction $f(x)$ (this can be done since you proved $PATH\in NC_2$) and send the result as input to the $NC_2$ circuit for $B$. This results in an $NC_2$ circuit for $A$.
-
$\begingroup$ Why do we require the subscript i in f(x) can't the transducer directly compute the entire output f(x)? $\endgroup$Vikash Balasubramanian– Vikash Balasubramanian2015年11月05日 19:45:48 +00:00Commented Nov 5, 2015 at 19:45
-
$\begingroup$ I wanted to talk about strictly logspace machines, and not ones who allow polynomial output tape. This also allows me to formulate the problem of computing $f(x)$ as inputs to $PATH,ドル since any bit of $f(x)$ is either 0 or 1, we can formulate this as a decision problem and talk about the usual configurations graph. $\endgroup$Ariel– Ariel2015年11月05日 19:50:58 +00:00Commented Nov 5, 2015 at 19:50
Explore related questions
See similar questions with these tags.